Divisibility and number theory in terms of a and bSimple Division ProofShow that if $x,y,z$ are positive integers, then $(xy + 1)(yz + 1)(zx + 1)$ is a perfect square iff $xy +1, yz +1, zx+1$ are all perfect squares.Find infinitely many pairs of integers $a$ and $b$ with $1 < a < b$, so that $ab$ exactly divides $a^2 +b^2 −1$Show that there are infinitely many pairs (a,b) such that both $x^2+ax+b$ and $x^2+2ax+b$ have integer rootsMaximal Consecutive Integer Sequencenumber of coprime divisors of n with their difference divisible by 3For any positive integer $a$, prove that there exists infinitely many composite $n$ such that $a^n-1equiv 1mod n$.

Bought a book that is in the public domain ... but the T&A of company says I can't redistribute it

Example of two undecidable languages that cannot be reduced to each other

how can I apply 3 shader by coordinates instead of unwrap UV?

Different colours for different sectors of piecewise function

Was Constantine The Great a Nicene Christian?

Does Estonia have discount supermarket chains like Aldi and Lidl?

How to assemble PCBs when SMT machine doesn't have enough feeders?

I might blow up!

Suppose I capture encrypted data that I want to decrypt. Could I use a server farm to decrypt?

Why do HK chefs use a white cloth to clutch wok?

Origin of Aliens in 'A Quiet Place'?

Can Teflon thread tape be reused?

Is there a word/phrase that can describe playing a musical instrument in a casual way?

Setting tack strip in concrete

What is the meaning of the Latin names of grammatical cases (in general, not in Latin)?

C - wrapping globals in a struct?

Too many pull requests in backlog

Do gray aliens exist in Star Trek?

How can medieval knights protects themselves against modern guns?

Why is it so important who the whistleblower in the Trump-Zelensky phone call is?

Exactly what color was the text on monochrome terminals with green-on-black and amber-on-black screens?

Can I call the airport to see if my boyfriend made it through customs?

Match blood types in C

Simulate reproduction in a population of oozes



Divisibility and number theory in terms of a and b


Simple Division ProofShow that if $x,y,z$ are positive integers, then $(xy + 1)(yz + 1)(zx + 1)$ is a perfect square iff $xy +1, yz +1, zx+1$ are all perfect squares.Find infinitely many pairs of integers $a$ and $b$ with $1 < a < b$, so that $ab$ exactly divides $a^2 +b^2 −1$Show that there are infinitely many pairs (a,b) such that both $x^2+ax+b$ and $x^2+2ax+b$ have integer rootsMaximal Consecutive Integer Sequencenumber of coprime divisors of n with their difference divisible by 3For any positive integer $a$, prove that there exists infinitely many composite $n$ such that $a^n-1equiv 1mod n$.






.everyoneloves__top-leaderboard:empty,.everyoneloves__mid-leaderboard:empty,.everyoneloves__bot-mid-leaderboard:empty
margin-bottom:0;

.everyonelovesstackoverflowposition:absolute;height:1px;width:1px;opacity:0;top:0;left:0;pointer-events:none;








5















$begingroup$


Are there infinitely many pairs of $(a, b)$ of relatively prime integers $a > 1$ and $b > 1$ such that $a^b+b^a$ is divisible by $a+b$?



I've spent almost two hours on this question to no avail. The small cases I have tried, such as $(3, 5)$, $(3, 7)$ and $(5, 7)$ all work, so I'm conjecturing that the answer is yes. However, all methods of proof I have tried have failed. Tried modulo arguments but can't really simplify my results.



Any help would be greatly appreciated.










share|cite|improve this question









$endgroup$










  • 2




    $begingroup$
    Can you show in detail an example/examples of your attempted proofs?
    $endgroup$
    – TheSimpliFire
    Sep 14 at 14:32

















5















$begingroup$


Are there infinitely many pairs of $(a, b)$ of relatively prime integers $a > 1$ and $b > 1$ such that $a^b+b^a$ is divisible by $a+b$?



I've spent almost two hours on this question to no avail. The small cases I have tried, such as $(3, 5)$, $(3, 7)$ and $(5, 7)$ all work, so I'm conjecturing that the answer is yes. However, all methods of proof I have tried have failed. Tried modulo arguments but can't really simplify my results.



Any help would be greatly appreciated.










share|cite|improve this question









$endgroup$










  • 2




    $begingroup$
    Can you show in detail an example/examples of your attempted proofs?
    $endgroup$
    – TheSimpliFire
    Sep 14 at 14:32













5













5









5


1



$begingroup$


Are there infinitely many pairs of $(a, b)$ of relatively prime integers $a > 1$ and $b > 1$ such that $a^b+b^a$ is divisible by $a+b$?



I've spent almost two hours on this question to no avail. The small cases I have tried, such as $(3, 5)$, $(3, 7)$ and $(5, 7)$ all work, so I'm conjecturing that the answer is yes. However, all methods of proof I have tried have failed. Tried modulo arguments but can't really simplify my results.



Any help would be greatly appreciated.










share|cite|improve this question









$endgroup$




Are there infinitely many pairs of $(a, b)$ of relatively prime integers $a > 1$ and $b > 1$ such that $a^b+b^a$ is divisible by $a+b$?



I've spent almost two hours on this question to no avail. The small cases I have tried, such as $(3, 5)$, $(3, 7)$ and $(5, 7)$ all work, so I'm conjecturing that the answer is yes. However, all methods of proof I have tried have failed. Tried modulo arguments but can't really simplify my results.



Any help would be greatly appreciated.







number-theory divisibility coprime






share|cite|improve this question













share|cite|improve this question











share|cite|improve this question




share|cite|improve this question










asked Sep 14 at 14:26









EthanMathsEthanMaths

534 bronze badges




534 bronze badges










  • 2




    $begingroup$
    Can you show in detail an example/examples of your attempted proofs?
    $endgroup$
    – TheSimpliFire
    Sep 14 at 14:32












  • 2




    $begingroup$
    Can you show in detail an example/examples of your attempted proofs?
    $endgroup$
    – TheSimpliFire
    Sep 14 at 14:32







2




2




$begingroup$
Can you show in detail an example/examples of your attempted proofs?
$endgroup$
– TheSimpliFire
Sep 14 at 14:32




$begingroup$
Can you show in detail an example/examples of your attempted proofs?
$endgroup$
– TheSimpliFire
Sep 14 at 14:32










2 Answers
2






active

oldest

votes


















4

















$begingroup$

Sil posted a now deleted almost correct answer, so this is based on those ideas. (The answer has since been undeleted and corrected)



Let $a=2k-1$, $b=2k+1$ for any integer $k$. $a,b$ are coprime. $a+b=4k$.



Note that
$$ab=4k^2-1 equiv -1 pmod4k,$$
$$a^2 = 4k^2 - 4k +1 equiv 1 pmod 4k,$$
$$b^2 = 4k^2 + 4k +1 equiv 1 pmod 4k.$$
Since $abequiv -1pmod4k$, $b^-1 equiv -apmod4k$.



Then
$$
a^b+b^a
=a^2k+1+b^2k-1
equiv
a^1 + b^-1
equiv
a-a
=0pmod4k.
$$



Thus $a^b+b^a equiv 0pmoda+b$, so $a+bmid a^b+b^a$.



Hence we have found infinitely many such pairs.






share|cite|improve this answer












$endgroup$














  • $begingroup$
    Nice, this is more elegant then my (now hopefully fixed) argument (I had to use binomial theorem to get from $2k$ modulus to $4k$).
    $endgroup$
    – Sil
    Sep 14 at 15:15











  • $begingroup$
    @Sil The binomial theorem argument is also pretty nice, (+1)
    $endgroup$
    – jgon
    Sep 14 at 15:18


















4

















$begingroup$

Sure, choose for example $a=2k-1,b=2k+1$ for any integer $k$. Clearly these pairs are coprime, and we have



beginalign
a^b+b^a&=(2k-1)^2k+1+(2k+1)^2k-1\
&equiv -1+(2k+1)2k+1+(2k-1)2ktag*\
&=8k^2\
&equiv 0 pmod4k.
endalign



where $(*)$ follows from Binomial theorem for example (or see jgon's answer for alternative argument). So $a^b+b^a equiv 0 bmod a+b$ since $a+b=4k$.






share|cite|improve this answer












$endgroup$
















    Your Answer








    StackExchange.ready(function()
    var channelOptions =
    tags: "".split(" "),
    id: "69"
    ;
    initTagRenderer("".split(" "), "".split(" "), channelOptions);

    StackExchange.using("externalEditor", function()
    // Have to fire editor after snippets, if snippets enabled
    if (StackExchange.settings.snippets.snippetsEnabled)
    StackExchange.using("snippets", function()
    createEditor();
    );

    else
    createEditor();

    );

    function createEditor()
    StackExchange.prepareEditor(
    heartbeatType: 'answer',
    autoActivateHeartbeat: false,
    convertImagesToLinks: true,
    noModals: true,
    showLowRepImageUploadWarning: true,
    reputationToPostImages: 10,
    bindNavPrevention: true,
    postfix: "",
    imageUploader:
    brandingHtml: "Powered by u003ca class="icon-imgur-white" href="https://imgur.com/"u003eu003c/au003e",
    contentPolicyHtml: "User contributions licensed under u003ca href="https://creativecommons.org/licenses/by-sa/4.0/"u003ecc by-sa 4.0 with attribution requiredu003c/au003e u003ca href="https://stackoverflow.com/legal/content-policy"u003e(content policy)u003c/au003e",
    allowUrls: true
    ,
    noCode: true, onDemand: true,
    discardSelector: ".discard-answer"
    ,immediatelyShowMarkdownHelp:true
    );



    );














    draft saved

    draft discarded
















    StackExchange.ready(
    function ()
    StackExchange.openid.initPostLogin('.new-post-login', 'https%3a%2f%2fmath.stackexchange.com%2fquestions%2f3356258%2fdivisibility-and-number-theory-in-terms-of-a-and-b%23new-answer', 'question_page');

    );

    Post as a guest















    Required, but never shown


























    2 Answers
    2






    active

    oldest

    votes








    2 Answers
    2






    active

    oldest

    votes









    active

    oldest

    votes






    active

    oldest

    votes









    4

















    $begingroup$

    Sil posted a now deleted almost correct answer, so this is based on those ideas. (The answer has since been undeleted and corrected)



    Let $a=2k-1$, $b=2k+1$ for any integer $k$. $a,b$ are coprime. $a+b=4k$.



    Note that
    $$ab=4k^2-1 equiv -1 pmod4k,$$
    $$a^2 = 4k^2 - 4k +1 equiv 1 pmod 4k,$$
    $$b^2 = 4k^2 + 4k +1 equiv 1 pmod 4k.$$
    Since $abequiv -1pmod4k$, $b^-1 equiv -apmod4k$.



    Then
    $$
    a^b+b^a
    =a^2k+1+b^2k-1
    equiv
    a^1 + b^-1
    equiv
    a-a
    =0pmod4k.
    $$



    Thus $a^b+b^a equiv 0pmoda+b$, so $a+bmid a^b+b^a$.



    Hence we have found infinitely many such pairs.






    share|cite|improve this answer












    $endgroup$














    • $begingroup$
      Nice, this is more elegant then my (now hopefully fixed) argument (I had to use binomial theorem to get from $2k$ modulus to $4k$).
      $endgroup$
      – Sil
      Sep 14 at 15:15











    • $begingroup$
      @Sil The binomial theorem argument is also pretty nice, (+1)
      $endgroup$
      – jgon
      Sep 14 at 15:18















    4

















    $begingroup$

    Sil posted a now deleted almost correct answer, so this is based on those ideas. (The answer has since been undeleted and corrected)



    Let $a=2k-1$, $b=2k+1$ for any integer $k$. $a,b$ are coprime. $a+b=4k$.



    Note that
    $$ab=4k^2-1 equiv -1 pmod4k,$$
    $$a^2 = 4k^2 - 4k +1 equiv 1 pmod 4k,$$
    $$b^2 = 4k^2 + 4k +1 equiv 1 pmod 4k.$$
    Since $abequiv -1pmod4k$, $b^-1 equiv -apmod4k$.



    Then
    $$
    a^b+b^a
    =a^2k+1+b^2k-1
    equiv
    a^1 + b^-1
    equiv
    a-a
    =0pmod4k.
    $$



    Thus $a^b+b^a equiv 0pmoda+b$, so $a+bmid a^b+b^a$.



    Hence we have found infinitely many such pairs.






    share|cite|improve this answer












    $endgroup$














    • $begingroup$
      Nice, this is more elegant then my (now hopefully fixed) argument (I had to use binomial theorem to get from $2k$ modulus to $4k$).
      $endgroup$
      – Sil
      Sep 14 at 15:15











    • $begingroup$
      @Sil The binomial theorem argument is also pretty nice, (+1)
      $endgroup$
      – jgon
      Sep 14 at 15:18













    4















    4











    4







    $begingroup$

    Sil posted a now deleted almost correct answer, so this is based on those ideas. (The answer has since been undeleted and corrected)



    Let $a=2k-1$, $b=2k+1$ for any integer $k$. $a,b$ are coprime. $a+b=4k$.



    Note that
    $$ab=4k^2-1 equiv -1 pmod4k,$$
    $$a^2 = 4k^2 - 4k +1 equiv 1 pmod 4k,$$
    $$b^2 = 4k^2 + 4k +1 equiv 1 pmod 4k.$$
    Since $abequiv -1pmod4k$, $b^-1 equiv -apmod4k$.



    Then
    $$
    a^b+b^a
    =a^2k+1+b^2k-1
    equiv
    a^1 + b^-1
    equiv
    a-a
    =0pmod4k.
    $$



    Thus $a^b+b^a equiv 0pmoda+b$, so $a+bmid a^b+b^a$.



    Hence we have found infinitely many such pairs.






    share|cite|improve this answer












    $endgroup$



    Sil posted a now deleted almost correct answer, so this is based on those ideas. (The answer has since been undeleted and corrected)



    Let $a=2k-1$, $b=2k+1$ for any integer $k$. $a,b$ are coprime. $a+b=4k$.



    Note that
    $$ab=4k^2-1 equiv -1 pmod4k,$$
    $$a^2 = 4k^2 - 4k +1 equiv 1 pmod 4k,$$
    $$b^2 = 4k^2 + 4k +1 equiv 1 pmod 4k.$$
    Since $abequiv -1pmod4k$, $b^-1 equiv -apmod4k$.



    Then
    $$
    a^b+b^a
    =a^2k+1+b^2k-1
    equiv
    a^1 + b^-1
    equiv
    a-a
    =0pmod4k.
    $$



    Thus $a^b+b^a equiv 0pmoda+b$, so $a+bmid a^b+b^a$.



    Hence we have found infinitely many such pairs.







    share|cite|improve this answer















    share|cite|improve this answer




    share|cite|improve this answer








    edited Sep 14 at 15:18

























    answered Sep 14 at 15:07









    jgonjgon

    20k3 gold badges25 silver badges48 bronze badges




    20k3 gold badges25 silver badges48 bronze badges














    • $begingroup$
      Nice, this is more elegant then my (now hopefully fixed) argument (I had to use binomial theorem to get from $2k$ modulus to $4k$).
      $endgroup$
      – Sil
      Sep 14 at 15:15











    • $begingroup$
      @Sil The binomial theorem argument is also pretty nice, (+1)
      $endgroup$
      – jgon
      Sep 14 at 15:18
















    • $begingroup$
      Nice, this is more elegant then my (now hopefully fixed) argument (I had to use binomial theorem to get from $2k$ modulus to $4k$).
      $endgroup$
      – Sil
      Sep 14 at 15:15











    • $begingroup$
      @Sil The binomial theorem argument is also pretty nice, (+1)
      $endgroup$
      – jgon
      Sep 14 at 15:18















    $begingroup$
    Nice, this is more elegant then my (now hopefully fixed) argument (I had to use binomial theorem to get from $2k$ modulus to $4k$).
    $endgroup$
    – Sil
    Sep 14 at 15:15





    $begingroup$
    Nice, this is more elegant then my (now hopefully fixed) argument (I had to use binomial theorem to get from $2k$ modulus to $4k$).
    $endgroup$
    – Sil
    Sep 14 at 15:15













    $begingroup$
    @Sil The binomial theorem argument is also pretty nice, (+1)
    $endgroup$
    – jgon
    Sep 14 at 15:18




    $begingroup$
    @Sil The binomial theorem argument is also pretty nice, (+1)
    $endgroup$
    – jgon
    Sep 14 at 15:18













    4

















    $begingroup$

    Sure, choose for example $a=2k-1,b=2k+1$ for any integer $k$. Clearly these pairs are coprime, and we have



    beginalign
    a^b+b^a&=(2k-1)^2k+1+(2k+1)^2k-1\
    &equiv -1+(2k+1)2k+1+(2k-1)2ktag*\
    &=8k^2\
    &equiv 0 pmod4k.
    endalign



    where $(*)$ follows from Binomial theorem for example (or see jgon's answer for alternative argument). So $a^b+b^a equiv 0 bmod a+b$ since $a+b=4k$.






    share|cite|improve this answer












    $endgroup$



















      4

















      $begingroup$

      Sure, choose for example $a=2k-1,b=2k+1$ for any integer $k$. Clearly these pairs are coprime, and we have



      beginalign
      a^b+b^a&=(2k-1)^2k+1+(2k+1)^2k-1\
      &equiv -1+(2k+1)2k+1+(2k-1)2ktag*\
      &=8k^2\
      &equiv 0 pmod4k.
      endalign



      where $(*)$ follows from Binomial theorem for example (or see jgon's answer for alternative argument). So $a^b+b^a equiv 0 bmod a+b$ since $a+b=4k$.






      share|cite|improve this answer












      $endgroup$

















        4















        4











        4







        $begingroup$

        Sure, choose for example $a=2k-1,b=2k+1$ for any integer $k$. Clearly these pairs are coprime, and we have



        beginalign
        a^b+b^a&=(2k-1)^2k+1+(2k+1)^2k-1\
        &equiv -1+(2k+1)2k+1+(2k-1)2ktag*\
        &=8k^2\
        &equiv 0 pmod4k.
        endalign



        where $(*)$ follows from Binomial theorem for example (or see jgon's answer for alternative argument). So $a^b+b^a equiv 0 bmod a+b$ since $a+b=4k$.






        share|cite|improve this answer












        $endgroup$



        Sure, choose for example $a=2k-1,b=2k+1$ for any integer $k$. Clearly these pairs are coprime, and we have



        beginalign
        a^b+b^a&=(2k-1)^2k+1+(2k+1)^2k-1\
        &equiv -1+(2k+1)2k+1+(2k-1)2ktag*\
        &=8k^2\
        &equiv 0 pmod4k.
        endalign



        where $(*)$ follows from Binomial theorem for example (or see jgon's answer for alternative argument). So $a^b+b^a equiv 0 bmod a+b$ since $a+b=4k$.







        share|cite|improve this answer















        share|cite|improve this answer




        share|cite|improve this answer








        edited Sep 14 at 15:14

























        answered Sep 14 at 14:58









        SilSil

        7,0562 gold badges19 silver badges46 bronze badges




        7,0562 gold badges19 silver badges46 bronze badges































            draft saved

            draft discarded















































            Thanks for contributing an answer to Mathematics Stack Exchange!


            • Please be sure to answer the question. Provide details and share your research!

            But avoid


            • Asking for help, clarification, or responding to other answers.

            • Making statements based on opinion; back them up with references or personal experience.

            Use MathJax to format equations. MathJax reference.


            To learn more, see our tips on writing great answers.




            draft saved


            draft discarded














            StackExchange.ready(
            function ()
            StackExchange.openid.initPostLogin('.new-post-login', 'https%3a%2f%2fmath.stackexchange.com%2fquestions%2f3356258%2fdivisibility-and-number-theory-in-terms-of-a-and-b%23new-answer', 'question_page');

            );

            Post as a guest















            Required, but never shown





















































            Required, but never shown














            Required, but never shown












            Required, but never shown







            Required, but never shown

































            Required, but never shown














            Required, but never shown












            Required, but never shown







            Required, but never shown









            Popular posts from this blog

            Tamil (spriik) Luke uk diar | Nawigatjuun

            Align equal signs while including text over equalitiesAMS align: left aligned text/math plus multicolumn alignmentMultiple alignmentsAligning equations in multiple placesNumbering and aligning an equation with multiple columnsHow to align one equation with another multline equationUsing \ in environments inside the begintabularxNumber equations and preserving alignment of equal signsHow can I align equations to the left and to the right?Double equation alignment problem within align enviromentAligned within align: Why are they right-aligned?

            Where does the image of a data connector as a sharp metal spike originate from?Where does the concept of infected people turning into zombies only after death originate from?Where does the motif of a reanimated human head originate?Where did the notion that Dragons could speak originate?Where does the archetypal image of the 'Grey' alien come from?Where did the suffix '-Man' originate?Where does the notion of being injured or killed by an illusion originate?Where did the term “sophont” originate?Where does the trope of magic spells being driven by advanced technology originate from?Where did the term “the living impaired” originate?